Difference between revisions of "2022 AIME I Problems/Problem 4"

(Solution 2)
(Solution 2)
Line 56: Line 56:
  
 
<math>
 
<math>
\begin{tabular}[t]{|l|ccccc|c|}
+
\begin{tabular}[t]{|l|ccc}
 
\multicolumn{7}{c}{Modular Table of r and s}\\\hline
 
\multicolumn{7}{c}{Modular Table of r and s}\\\hline
Name&\#1&\#2&\#3&\#4&\#5&Total\\\hline
+
Name&\#1&\#2&\#3\\\hline
John Doe&5&5&3&2&1&16\\
+
John Doe&5&5&3&\\
Jane Doe&5&5&5&4&5&24\\
+
Jane Doe&5&5&5\\
Richard Feynman&5&5&5&5&5&25\\\hline
+
Richard Feynman&5&5&5\\\hline
 
\end{tabular}
 
\end{tabular}
 
</math>
 
</math>

Revision as of 23:15, 17 February 2022

Problem

Let $w = \dfrac{\sqrt{3} + i}{2}$ and $z = \dfrac{-1 + i\sqrt{3}}{2},$ where $i = \sqrt{-1}.$ Find the number of ordered pairs $(r,s)$ of positive integers not exceeding $100$ that satisfy the equation $i \cdot w^r = z^s.$

Solution

We rewrite $w$ and $z$ in polar form: \begin{align*} w &= e^{i\cdot\frac{\pi}{6}}, \\ z &= e^{i\cdot\frac{2\pi}{3}}. \end{align*} The equation $i \cdot w^r = z^s$ becomes \begin{align*} e^{i\cdot\frac{\pi}{2}} \cdot \left(e^{i\cdot\frac{\pi}{6}}\right)^r &= \left(e^{i\cdot\frac{2\pi}{3}}\right)^s \\ e^{i\left(\frac{\pi}{2}+\frac{\pi}{6}r\right)} &= e^{i\left(\frac{2\pi}{3}s\right)} \\ \frac{\pi}{2}+\frac{\pi}{6}r &= \frac{2\pi}{3}s+2\pi k \\ 3+r &= 4s+12k \\ 3+r &= 4(s+3k). \end{align*} for some integer $k.$

Since $4\leq 3+r\leq 103$ and $4\mid 3+r,$ we conclude that \begin{align*} 3+r &\in \{4,8,12,\ldots,100\}, \\ s+3k &\in \{1,2,3,\ldots,25\}. \end{align*} Note that the values for $s+3k$ and the values for $r$ have one-to-one correspondence.

We apply casework to the values for $s+3k:$

  1. $s+3k\equiv0\pmod{3}$
  2. There are $8$ values for $s+3k,$ so there are $8$ values for $r.$ It follows that $s\equiv0\pmod{3},$ so there are $33$ values for $s.$

    There are $8\cdot33=264$ ordered pairs $(r,s)$ in this case.

  3. $s+3k\equiv1\pmod{3}$
  4. There are $9$ values for $s+3k,$ so there are $9$ values for $r.$ It follows that $s\equiv1\pmod{3},$ so there are $34$ values for $s.$

    There are $9\cdot34=306$ ordered pairs $(r,s)$ in this case.

  5. $s+3k\equiv2\pmod{3}$
  6. There are $8$ values for $s+3k,$ so there are $8$ values for $r.$ It follows that $s\equiv2\pmod{3},$ so there are $33$ values for $s.$

    There are $8\cdot33=264$ ordered pairs $(r,s)$ in this case.

Together, the answer is $264+306+264=\boxed{834}.$

~MRENTHUSIASM

Solution 2

First we recognize that $w = cis(30^{\circ})$ and $z = cis(12^{\circ})$ because the cosine and sine sums of those angles give the values of $w$ and $z$, respectively. By Demoivre's theorem, $cis(\theta)^n = cis(n\theta)$. When you multiply by $i$, we can think of that as rotating the complex number 90 degrees counterclockwise in the complex plane. Therefore, by the equation we know that $30r + 90$ and $120s$ land on the same angle.

This means that:

\[30r + 90 \equiv 120s \pmod{360}\]

Which we can simplify to

\[r+3 \equiv 4s \pmod{12}\].

$\begin{tabular}[t]{|l|ccc} \multicolumn{7}{c}{Modular Table of r and s}\\\hline Name&\#1&\#2&\#3\\\hline John Doe&5&5&3&\\ Jane Doe&5&5&5\\ Richard Feynman&5&5&5\\\hline \end{tabular}$ (Error compiling LaTeX. Unknown error_msg)

Video Solution (Mathematical Dexterity)

https://www.youtube.com/watch?v=XiEaCq5jf5s

See Also

2022 AIME I (ProblemsAnswer KeyResources)
Preceded by
Problem 3
Followed by
Problem 5
1 2 3 4 5 6 7 8 9 10 11 12 13 14 15
All AIME Problems and Solutions

The problems on this page are copyrighted by the Mathematical Association of America's American Mathematics Competitions. AMC logo.png